Q6

 
rachael.swetnam
Thanks Received: 0
Forum Guests
 
Posts: 6
Joined: November 30th, 2010
 
 
 

Q6

by rachael.swetnam Tue Nov 30, 2010 11:15 pm

Hi,
Thanks so much for everything you've posted! I cannot seem to figure out #6. Even the diagram is not helping.

Thanks :)
User avatar
 
bbirdwell
Thanks Received: 864
Atticus Finch
Atticus Finch
 
Posts: 803
Joined: April 16th, 2009
 
 
 

Re: PT33, S4, G2 - Birds in a Forest

by bbirdwell Sat Dec 04, 2010 5:53 pm

It's important here to acknowledge the phrase "complete and accurate." This essentially means, "For which one of these choices can the birds noted be OUT and ALL the other birds be IN?"

Let's take each constraint one by one and eliminate choices.

1. H --> ~G.

This eliminates (E). If H is not in the choice, that means it's IN, and if it's IN, G must be OUT. However, G is not in the choice, either, so that means G is in, which violates the rule.

2. J or M --> H, contrapositive: ~H --> ~J and ~M. Using the contrapositive, if H is in the choice, J and M must also be in the choice. This eliminates (B).

3. W --> G; ~G --> ~W. So if G is in the choice, W must be in the choice. This eliminates (C).

4. ~J --> S; ~S --> J. This essentially means that only one of J or S can be in the choice -- they cannot both be out. This eliminates (A).
I host free online workshop/Q&A sessions called Zen and the Art of LSAT. You can find upcoming dates here: http://www.manhattanlsat.com/zen-and-the-art.cfm